LSAT and Law School Admissions Forum

Get expert LSAT preparation and law school admissions advice from PowerScore Test Preparation.

 Adam Tyson
PowerScore Staff
  • PowerScore Staff
  • Posts: 5153
  • Joined: Apr 14, 2011
|
#21259
Hey Sherry, let me see if I can help. First things first here - the question is not a Could Be True question but rather a Cannot Be True, in that it says that each of the following could be true EXCEPT. So, which of these answer does NOT work? In other words, which answer cannot be the complete list of everything Groups S and Y have in common?

Start by looking at the numerical distribution in the game, which they are kind enough to spell out for you with the first 4 rules. Group R has just 1 variable, Groups S and T have two each, and Group Y has three.

Next, since neither P nor L can be in the same group as G, we can infer that G is NOT among the variables in Group Y. That group has to be H, L and P.

Also, while it doesn't end up impacting this particular question, G cannot be in Group R either, because the last rule requires that anywhere G goes, H must also go.

Now, think about each answer in q9 and, in the cases of answers A and B ask yourself "what could the other variable in Group S be?" It can't be anything else that appears in Group Y, because then we wouldn't have a complete list of the variables that those two groups have in common.

For answer A, the other variable in Group S could be G. That is, Group S could have H and G in it while Group Y has HLP in it, and the only variable in both is H. Answer A therefore could be true, and is therefore a wrong answer.

What about answer B now? If P is in Group S, what could you pair it with? Not G - that violates one of the rules. If we tried to pair it with H or L, then P would NOT be the only variable that is in both S and Y. That's why B is the correct answer.

Hope that cleared things up!
 LawLover
  • Posts: 29
  • Joined: Mar 26, 2018
|
#46896
Why is the answer to question #9 not B? This is where I started to give up on this game. I tried all the answers think they all are okay, and I still do not understand how the answer is B.
 Adam Tyson
PowerScore Staff
  • PowerScore Staff
  • Posts: 5153
  • Joined: Apr 14, 2011
|
#47027
See the discussions above here, LawLover, and also at this link where we discuss it in detail: lsat/viewtopic.php?t=6212

We are looking for an answer that CANNOT be the list of everything that groups S and Y have in common. P cannot be the only thing they have in common, because there would also have to be either H or L in group S. The only other alternative would be for S to have PG in it, and that would violate the rule that says they cannot be in the same group!
 LawLover
  • Posts: 29
  • Joined: Mar 26, 2018
|
#47045
Thanks for the reference Adam Tyson. The previous discussions that I looked at really made more sense, and I understood it better.

Thanks
LawLover
 Peter8123
  • Posts: 2
  • Joined: Jul 10, 2018
|
#47784
Adam,

I'm sorry for bringing up an old and possibly already answered post, but even after reading all of the replies on this page, as well as others, I still have not come across the specific problem I am finding with the answer of B for question 9, and the reasoning behind this answer.

I am using the LG Bible 2016 Edition. On page 355, the explanation states that:

"Since H, L, and P each learn Yoruba, it seems likely that any combination of the two of those three researchers could learn Swahili. Since answer choices (C), (D), and (E) each list two of H, L, or P, it seems unlikely that any of those answer choices are correct. Thus, let us focus on answer choices (A) and (B)." This is apparently all of the information required to exclude answer choices (C), (D), and (E), as no mention of these answer choices are made after the above quote.

Continuing the explanation, on page 356, in regards to answer choices (A) and (B), they state:

"In this instance the second researcher who learns Swahili cannot be H or L since neither is listed in the answer choice... Thus P cannot be the only person who learns both Swahili and Yoruba and answer choice (B) is correct."

I agree that any two combination of H, L, and P could learn Swahili. What I do not agree with is that any two combination of H, L, and P could also learn Yoruba, as Yoruba requires THREE researchers. Therefore, every single answer choice (A) through (D) has researchers not "listed in the answer choice," as the quote on page 356 states is enough to disqualify answer choice (A).

The way I currently see it, none of the answer choices are complete. Any help would be greatly appreciated.
 Adam Tyson
PowerScore Staff
  • PowerScore Staff
  • Posts: 5153
  • Joined: Apr 14, 2011
|
#47799
Happy to help, Peter! The list we are looking at in this answer choice is a complete and accurate list of the researchers that could be in both of those groups, S and Y, in any one particular scenario. You can never have all three in group S, which is limited to two researchers.

H and L COULD be the complete and accurate list of the researchers that the two groups have in common, if H and L were the two researchers that learn S. Same thing for H and P, and for P and L. H can be the only one in common if H and G are the ones to learn S.

P, however, cannot be the complete list of researchers the two have in common, because group S would have to also include either H or L, such that both P and the other one would be in groups S and Y.

They key here is that we are looking at a single solution, not all the various combinations that could exist. That should do the trick for you, I expect! The language "could be a complete and accurate list" indicates "in a given solution"; if instead it said "is a complete and accurate list of all the researchers that could" be in both groups, then you would need all three of H, L, and P. Crucial difference, "is...all" vs "could be". Watch out for that!
 Peter8123
  • Posts: 2
  • Joined: Jul 10, 2018
|
#47820
Adam,

Thank you for clearing that up. It makes complete sense now.

Best,

Peter
 Mastering_LSAT
  • Posts: 35
  • Joined: Jul 30, 2020
|
#81956
Hello,

After reading numerous explanations, I still can't understand why it is okay for the answer choice (A) to bring Geologist and complete the list with Historian, but it is not okay for the answer choice (B) to bring Linguist or Historian to complete the list with Paleontologist?

Thank you!
 Adam Tyson
PowerScore Staff
  • PowerScore Staff
  • Posts: 5153
  • Joined: Apr 14, 2011
|
#82376
The question is asking about the COMPLETE list that the two groups could have in common, so if group S had P and L or P and S, then groups S and Y would have BOTH of those in common, and P, by itself, would not be the complete list but only a partial list.

For answer A, if group S had H and G in it, then groups S and Y would have ONLY H in common, because G is not in group Y. That's why A works, and is therefore a wrong answer to this EXCEPT question.
User avatar
 Owl224
  • Posts: 6
  • Joined: Nov 09, 2021
|
#93961
Hi! I feel like I’m missing a really key fundamental somewhere because I did not make the connection for that the G —> H conditional would create a GH block. How does this work? I thought we only get positive blocks if it was a double arrow (G<—>H)?

Thank you!

Get the most out of your LSAT Prep Plus subscription.

Analyze and track your performance with our Testing and Analytics Package.